Side 1 av 1

Derivert

Lagt inn: 26/01-2014 19:48
av Gjest
Hei, jeg har en en derviasjonsoppgave her, som jeg har løst, men jeg får et lite avvik fra fasiten.

Oppgaven:

[tex]\sqrt{(x^2-2)^3-4)}[/tex]

Jge poster bare min svar, siden det tar lang tid å plotte inn i tex editor:

Mitt endelige svar: [tex]\frac{3x(x^2-2)^2}{\sqrt{(x^2-2)^3-4}}[/tex]

Fasiten sier: [tex]\frac{3x(x^2-2)^2}{\sqrt{(x-2)^3-4}}[/tex]

Hvem har da egentlig rett? :P

Re: Derivert

Lagt inn: 26/01-2014 20:39
av Kork
Det er greit å bruke en kalkulator som f.eks http://www.wolframalpha.com/ for å sjekke svarene dine.

Ellers er det greit å bare ta små steg om gangen, slik som her:

[tex]\eqalign{ & \frac{d}{{dx}}\sqrt {{{\left( {{x^2} - 2} \right)}^3} - 4} \cr & = \frac{d}{{dx}}{\left( {{{\left( {{x^2} - 2} \right)}^3} - 4} \right)^{\frac{1}{2}}} \cr & = \frac{1}{2}{\left( {{{\left( {{x^2} - 2} \right)}^3} - 4} \right)^{ - \frac{1}{2}}}\frac{d}{{dx}}\left( {{{\left( {{x^2} - 2} \right)}^3} - 4} \right) \cr & = \frac{1}{2}{\left( {{{\left( {{x^2} - 2} \right)}^3} - 4} \right)^{ - \frac{1}{2}}}\left( {\frac{d}{{dx}}{{\left( {{x^2} - 2} \right)}^3} - \frac{d}{{dx}}4} \right) \cr & = \frac{1}{2}{\left( {{{\left( {{x^2} - 2} \right)}^3} - 4} \right)^{ - \frac{1}{2}}}\left( {3{{\left( {{x^2} - 2} \right)}^2}\frac{d}{{dx}}\left( {{x^2} - 2} \right) - 0} \right) \cr & = \frac{1}{2}{\left( {{{\left( {{x^2} - 2} \right)}^3} - 4} \right)^{ - \frac{1}{2}}}\left( {3{{\left( {{x^2} - 2} \right)}^2}2x} \right) \cr & = \frac{{3x{{\left( {{x^2} - 2} \right)}^2}}}{{\sqrt {{{\left( {{x^2} - 2} \right)}^3} - 4} }} \cr}[/tex]

Re: Derivert

Lagt inn: 26/01-2014 22:14
av Gjest
Mhm, gjorde det faktisk tidligere, men det er da fasiten som er feil? og jeg som er da rett? Siden wolfram gir samme svar som meg, og andre programmer som jeg har prøvd.

Re: Derivert

Lagt inn: 26/01-2014 23:22
av Kork
Ja fasit er feil